Question

Let {X1, ..., Xn} be i.i.d. from a distribution with pdf f(x; θ) = θ/xθ+1 for...

Let {X1, ..., Xn} be i.i.d. from a distribution with pdf f(x; θ) = θ/xθ+1 for θ > 2 and x > 1.

(a) (10 points) Calculate EX1 and V ar(X1).

(b) (5 points) Find the method of moments estimator of θ.

(c) (5 points) If we denote the method of moments estimator as ˆθ1. What does √ n( ˆθ1 − θ) converge in distribution to? (d) (5 points) Is the method of moment estimator efficient? Verify your answer.

Homework Answers

Know the answer?
Your Answer:

Post as a guest

Your Name:

What's your source?

Earn Coins

Coins can be redeemed for fabulous gifts.

Not the answer you're looking for?
Ask your own homework help question
Similar Questions
Let X1,...,Xn be a random sample from the pdf f(x;θ) = θx^(θ−1) , 0 ≤ x...
Let X1,...,Xn be a random sample from the pdf f(x;θ) = θx^(θ−1) , 0 ≤ x ≤ 1 , 0 < θ < ∞ Find the method of moments estimator of θ.
Let X1. . . . Xn be i.i.d f(x; θ) = θ(1 − θ)^x x =...
Let X1. . . . Xn be i.i.d f(x; θ) = θ(1 − θ)^x x = 0, . . .Find the UMVUE of (1−θ) θ if such exists
Let X1. . . . Xn be i.i.d f(x; θ) = θ(1 − θ)^x x =...
Let X1. . . . Xn be i.i.d f(x; θ) = θ(1 − θ)^x x = 0, . . .Find the UMVUE of (1−θ)/ θ if such exists
Let X1, . . . , Xn be i.i.d from pmf f(x|λ) where f(x) = (e^(−λ)*(λ^x))/x!,...
Let X1, . . . , Xn be i.i.d from pmf f(x|λ) where f(x) = (e^(−λ)*(λ^x))/x!, λ > 0, x = 0, 1, 2 a) Find MoM (Method of Moments) estimator for λ b) Show that MoM estimator you found in (a) is minimal sufficient for λ c) Now we split the sample into two parts, X1, . . . , Xm and Xm+1, . . . , Xn. Show that ( Sum of Xi from 1 to m, Sum...
Let X1, X2, . . . , Xn be iid random variables with pdf f(x|θ) =...
Let X1, X2, . . . , Xn be iid random variables with pdf f(x|θ) = θx^(θ−1) , 0 < x < 1, θ > 0. Is there an unbiased estimator of some function γ(θ), whose variance attains the Cramer-Rao lower bound?
Let X1,..., Xn be a random sample from a distribution with pdf as follows: fX(x) =...
Let X1,..., Xn be a random sample from a distribution with pdf as follows: fX(x) = e^-(x-θ) , x > θ 0 otherwise. Find the sufficient statistic for θ. Find the maximum likelihood estimator of θ. Find the MVUE of θ,θˆ Is θˆ a consistent estimator of θ?
Let X1,...,Xn be iid exp(θ) rvs. (a) Compute the pdf of Xmin. I have the pdf...
Let X1,...,Xn be iid exp(θ) rvs. (a) Compute the pdf of Xmin. I have the pdf (b) Create an unbiased estimator for θ based on Xmin. Compute the variance of the resulting estimator. (c) Perform a Monte Carlo simulation of N= 10,0000 samples of your unbiased estimator from part (b) using θ = 2 and n = 100 to validate your answer. Include a histogram of the samples. (d) Which is more efficient: your estimator from part (b) or the...
Let X1, X2 · · · , Xn be a random sample from the distribution with...
Let X1, X2 · · · , Xn be a random sample from the distribution with PDF, f(x) = (θ + 1)x^θ , 0 < x < 1, θ > −1. Find an estimator for θ using the maximum likelihood
Let X1, . . . , Xn be a random sample from the following pdf: f(x|θ)=...
Let X1, . . . , Xn be a random sample from the following pdf: f(x|θ)= (x/θ)*e^(-x^2/2θ). x>0 (a) Find a sufficient statistic for θ.
Consider a random sample X1, X2, ⋯ Xn from the pdf fx;θ=.51+θx, -1≤x≤1;0, o.w., where (this...
Consider a random sample X1, X2, ⋯ Xn from the pdf fx;θ=.51+θx, -1≤x≤1;0, o.w., where (this distribution arises in particle physics). Find the method of moment estimator of θ. Compute the variance of your estimator. Hint: Compute the variance of X and then apply the formula for X, etc.
ADVERTISEMENT
Need Online Homework Help?

Get Answers For Free
Most questions answered within 1 hours.

Ask a Question
ADVERTISEMENT